Making three equations orthonormal by forcing the constants

  • Thread starter Thread starter pchemstudent
  • Start date Start date
  • Tags Tags
    Constants
Click For Summary
The discussion focuses on making three equations orthonormal over the interval [-1, 1] by determining the coefficients a, b, c, d, e, and f. The definition of orthonormality is clarified, stating that functions must satisfy specific integral conditions to be orthonormal. It is suggested that the first function, f1(x), equals a = 1/√2, while the second function leads to the conclusion that b must equal 0 and c can be ±√(3/2). The conversation emphasizes the need to apply integrals to verify orthonormality for the second and third functions. Participants encourage further exploration of the problem to complete the solution.
pchemstudent
Messages
2
Reaction score
0
1. Make these three equations orthonormal to each other in the interval 1 less than or equal to x greater than or equal to 1 (this may be a typo as I think it should read 1 less than or equal to x less than or equal to one) by determining the appropriate values for the coefficients a b c d e f.

1) a
2) b+c*x
3) d+e*x+f*x^2


Homework Equations


Definition of orthonormality is all we are given.


The Attempt at a Solution


I think I understand how orthonormality works with vectors but this equation thing is throwing me off. Perhaps if someone could start by getting my on the right track then I can complete it (as it is a homework problem). Thanks!

Erik
 
Physics news on Phys.org
sorry. the paper says -1 less than or equal to x greater than or equal to 1 but I think it should be -1 less than or equal to x less than or equal to 1... as in [-1,1]
 
pchemstudent said:
1. Make these three equations orthonormal to each other in the interval -1 less than or equal to x greater than or equal to 1 by determining the appropriate values for the coefficients a b c d e f.

1) a
2) b+c*x
3) d+e*x+f*x^2

Homework Equations


Definition of orthonormality is all we are given.

The Attempt at a Solution


I think I understand how orthonormality works with vectors but this equation thing is throwing me off. Perhaps if someone could start by getting my on the right track then I can complete it (as it is a homework problem). Thanks!

Erik

Hi Eric.

You could edit your original post to avoid confusion.

What's your definition of orthonormality?

A set of functions, {fi(x) | i ∈ N}, is orthonormal on some interval [a, b] if:

\int_a^b f_i(x)\cdot f_j(x)\,dx=\left\{\begin{array}{cc}\ 1,&amp;\mbox{ if }<br /> i=j\\ \ 0, &amp; \mbox{ if } i\neq j\end{array}\right.

In this case, [a, b] = [-1, 1].

Therefore, do something like:

Let f1(x)=a.
Let f2(x)=b+c·x.
Let f3(x)=d+e·x+f·x2.

Clearly, a = 1/√(2).

\int_{-1}^1 (b+c\cdot x)^2 dx=1

b^2\int_{-1}^1 (1+\frac{2c}{b} x+\frac{c^2}{b^2}\,x^2) dx=1

\left(x+\frac{2c}{2b} x^2+\frac{c^2}{3b^2}\,x^3\right)_{x=-1}^{x=1} =\frac{1}{b^2}

2 + \frac{2c^2}{3b^2} =\frac{1}{b^2}

6\,b^2+2\,c^2 =3

Also,
\int_{-1}^1 (\frac{1}{\sqrt{2}})(b+c\cdot x) dx=0

\int_{-1}^1 (\frac{1}{\sqrt{2}})(b+c\cdot x) dx=0

\left(b\,x+\frac{c}{2} x^2\right)_{x=-1}^{x=1} =0

\displaystyle 2\,b=0\quad\to\quad b=0\,

Then:
c=\pm\sqrt{\frac{3}{2}}

Now, continue on with it.
 
Last edited:
Question: A clock's minute hand has length 4 and its hour hand has length 3. What is the distance between the tips at the moment when it is increasing most rapidly?(Putnam Exam Question) Answer: Making assumption that both the hands moves at constant angular velocities, the answer is ## \sqrt{7} .## But don't you think this assumption is somewhat doubtful and wrong?

Similar threads

  • · Replies 16 ·
Replies
16
Views
3K
  • · Replies 11 ·
Replies
11
Views
3K
  • · Replies 4 ·
Replies
4
Views
2K
  • · Replies 3 ·
Replies
3
Views
2K
  • · Replies 22 ·
Replies
22
Views
3K
Replies
9
Views
2K
  • · Replies 2 ·
Replies
2
Views
1K
Replies
4
Views
2K
  • · Replies 1 ·
Replies
1
Views
998
  • · Replies 7 ·
Replies
7
Views
2K